which is the right equation

Answers

Answer 1

Answer:

C

Step-by-step explanation:

The equation of a line in slope- intercept form is

y = mx + c ( m is the slope and c the y- intercept )

y = [tex]\frac{2}{3}[/tex] x + 1 ← is in slope- intercept form

with slope m = [tex]\frac{2}{3}[/tex]

Parallel lines have equal slopes , then

the slope of a parallel rail has slope = [tex]\frac{2}{3}[/tex]

The only equation with this slope is

y = [tex]\frac{2}{3}[/tex] x + 3 ← could represent the path of the second rail → C


Related Questions

Write the gradient and the y intercept of the line y=-2x +4

Answers

Answer:

[tex]{ \bf{gradient = { \boxed{ - 2}}}} \\ \\ { \bf{y - intercept = { \boxed{4}}}}[/tex]

Simplify. Rewrite the expression in the form x^n (x^2)^{4}

Answers

Answer:

x^8

Step-by-step explanation:

(x^2)^{4}

We know a^b^c = a^(b*c)

x^(2*4)

x^8

Answer:

x^8 <33

Step-by-step explanation:


[tex]4 - \frac{2}{3}x = \frac{x - 6}{5} [/tex]
x=? Please tell​

Answers

4-2/3x=x-6/5

Answer: X=6

Is 9/72 rational or irrational

Answers

i think answer is rational

Answer:

Rational

Step-by-step explanation:

The Numerator and Denominator is a whole number and its a fraction

there are two pots of milk containing 20 liters and 24 liters respectively .find the capacity of the largest container that can measure exactly milk both the cans​

Answers

Answer:

A can with capacity 44 litres

Step-by-step explanation:

If you like my answer than please mark me brainliest thanks

helphelphelphelphelphelp

Answers

Answer:

E - (8,-9)

F - (9,-9)

G - (9,-4)

Step-by-step explanation:

The first number in the bracket relates to the x-axis

The second number relates to the y-axis

If I start a road trip with 10,425 miles on my auto meter and I end it with 23,840 miles showing how many miles was my trip

Answers

Answer:

13,415 miles

Step-by-step explanation:

Final position - initial position = Miles traveled:

23,840 miles - 10,425 miles = 13,415 miles

Ten tiles numbered 1 through 10 are turned face down. One tile is turned up at random, and a die is rolled and the number is square. What is the probability, written as a percent, that the product of the numbers on the tile and the die will be a perfect cube?

Answers

Answer:

18.33333333%. Please mark as brainliest if I’m Correct. :)

Step-by-step explanation:

Help anyone can help me do the question,I will mark brainlest.​

Answers

Answer:

Step-by-step explanation:

S=pi*R^2=(area of the sector)*(360/angle of AOB)

so 22/7*9^2=99*360/angle of AOB

-> AOB=140

perimeter of circle: 2*pi*R

permiter of sector : 2*pi*140/360*R+ 2R=22+9*2=40

For there is always light, if only we’re brave enough to see it.

If only we’re brave enough to be it.

Answers

Answer:

sun is the answer to absolutely

69°
X/75°
36°
X =
degrees
Enter

Answers

Answer:

x = 105°

Step-by-step explanation:

The exterior angle of a triangle is equal to the sum of the 2 opposite interior angles

x is an exterior angle of the triangle , then

x = 69° + 36° = 105°

MUST SHOW WORK.
please do this ASAP!
This is about numerical expressions!

Answers

[tex]{\huge\fcolorbox{blue}{black}{\pink{\:\:\:\:\:\:\:\:\:\:\:\:\:\:\: Solution\:\:\:\:\:\:\:\:\:\:\:\:\:\:\:}}}[/tex]

[tex]2(4 - 1) {}^{2} - 3 + 2(5) \\ \\ = 2(3) {}^{2} - 3 + 2(5) \\ \\ = 2 \times 9 - 3 + 2(5) \\ \\ = 18 - 3 +2(5)\\ \\ =>15-10 = 25[/tex]

Hope This Helps You

[tex]\huge\text{Hey there!}[/tex]

[tex]\mathsf{2(4 - 1)^2 - 3 + 2(5)}[/tex]

[tex]\mathsf{4 - 1 = \bf 3}[/tex]

[tex]\mathsf{= 2(3^2) - 3 + 2(5)}[/tex]

[tex]\mathsf{3^2}\\\mathsf{= 3\times3}\\\mathsf{= \bf 9}[/tex]

[tex]\mathsf{= 2(9) - 3 + 2(5)}[/tex]

[tex]\mathsf{2(9)}\\\mathsf{= \bf 18}[/tex]

[tex]\mathsf{= 18 - 3 + 2(5)}[/tex]

[tex]\mathsf{18 - 3}\\\mathsf{= \bf 15}[/tex]

[tex]\mathsf{= 15 + 2(5)}[/tex]

[tex]\mathsf{2(5)}\\\mathsf{= \bf 10}[/tex]

[tex]\mathsf{= 15 + 10}\\\mathsf{= \bf 25}[/tex]

[tex]\boxed{\boxed{\huge\text{Therefore, your ANSWER is: \textsf{25}}}}\huge\checkmark[/tex]

[tex]\huge\text{Good luck on your assignment \& enjoy your day! }[/tex]

~[tex]\frak{Amphitrite1040:)}[/tex]

please answer and no links please ....​

Answers

Step-by-step explanation:

ℎ = -4, -3×(-4)+2 = 12+2 = 14

ℎ = -2, -3×(-2)+2 = 6+2 = 8

ℎ = 0, -3×0+2 = -3+2 = 2

ℎ = 3, -3×3+2 = -9+2 = -7

ℎ = 5, -3×5+2 = -15+2 = -13

What is the value of the expression 3.9x10^5/1.3x10^2?

I will mark Brainliest whoever person I actually trust.

Answers

Answer:

3 * 10 ^3

Step-by-step explanation:

Divide the numbers

3.9 / 1.3

3

Subtract the powers of 10

10^5 / 10^2 = 10 ^ (5-2) = 10^3

The result is 3 * 10 ^3

[tex] \frac{3.9 \times {10}^{5} }{1.3 \times {10}^{2} } \\ = \frac{3.9}{1.3} \times {10}^{5 - 2} \\ = 3.0 \times {10}^{3} [/tex]

Hope you could get an idea from here.

Doubt clarification - use comment section.

Find the number of bit strings that satisfies the given conditions. The bit strings of length 11 having at least four 1s

Answers

Answer:

Step-by-step explanation:

Write the digits used in binary number system

Answers

Answer:

Binary number is a number expressed in the base-2 numeral system or binary numeral system, a method of mathematical expression which uses only two symbols: typically "0" (zero) and "1" (one). The base-2 numeral system is a positional notation with a radix of 2.

Factor the expression
Thank you so muchh

Answers

Answer:

(x+8)(x-5)

Step-by-step explanation:

x^2 +3x-40

What two numbers multiply to -40 and add to 3

8 * -5 = -40

8+-5 = 3

(x+8)(x-5)

Answer:

(x + 8)(x - 5)

Brainliest, please!

Step-by-step explanation:

(x + )(x - )

Factors of 40: 1 and 40, 2 and 20, 4 and 10, 5 and 8

(x + 8)(x -5)

Find the length of the side labeled 2. Round intermediate values to
the nearest tenth. Use the rounded values to calculate the next
value. Round your final answer to the nearest tenth. PLEASE HELP ASAP!

A. 11.6
B. 13
C. 16.6
D. 22.4

Answers

Answer:

B

Step-by-step explanation:

The perpendicular is sin(43)*39=21.82. Next tan(32)=x/21.82. x=21.82*tan(32)=13.6

The value of x in the given question is 16.6

What are trigonometric ratios in a right angle triangle ?

A right angle triangle has 3 sides opposite is the side opposite to the angle formed by two adjacent sides.hypotenuse is the largest side and adjacent is the remaining side.

Assuming height of the triangle = y

Sin∅ = opposite/hypotenuse

sin43° = y/39

y= 39sin43°

y = 26.6
From this we will solve the value of x

tan∅ = opposite/adjacent

here adjacent = 26.6 and opposite = x

tan32° = x/26.6

x = 26.6tan32°

x = 16.6

Learn more about Trigonometric ratios here :

https://brainly.com/question/14977354

#SPJ5

One number is three times another number. If the sum of the numbers is 36, determine the two numbers.

Answers

Step-by-step explanation:

A=3B

A+B=36

3B+B=36

4B=36

B=36/4

Step-by-step explanation:

let x represent the smaller number

3x is the second number

x+3x=36

4x=36

4x/4=36/4

x=9

the second number is 3(9)

=27

giá tiền cước của một hãng taxi được tính như sau :giá mở đến 1km đầu là 12000 đồng ; 10500 đồng/km cho 20 km tiếp theo và 99000 đồng/km cho những km còn lại . Hỏi ông sáu đi từ nhà đến khu du lịch sinh thái Bò Cạp Vàng với quãng đường 62km bằng taxi của hãng đó hết bao nhiêu

Answers

Answer:

Ông sáu sẽ mất 4281000 đồng

log 55 is the power to which ---- must be raised in order to produce a value of 55

Answers

Answer:

10

Step-by-step explanation:

just took the quiz

ASAP!!! PLEASE

A pair of equations is shown below: y = 2x − 1 y = 4x − 5 Part A: In your own words, explain how you can solve the pair of equations graphically. Write the slope and y-intercept for each equation that you will plot on the graph to solve the equations. (6 points) Part B: What is the solution to the pair of equations? (4 points)

Answers

Step-by-step explanation:

526 133 3821

P: 12345

j _o _I _n g _I _r l

o. n z-oo-o-m

What is the volume of the solid? Let π=3.14
.

Answers

The volume of the solid as shown in the task content is; 2786.2cm³.

What is the volume of the solid?

It can be obtained from the task content that the solid given is a Cone.

On this note, the volume of the cone can be evaluated by means of the formula;

V = (1/3)π r²h where(r =22/2 = 11 and h=22).

Hence, Volume, V = (1/3) ×3.14× (11)²×22 = 2786.2cm³.

Read more on volume;

https://brainly.com/question/13677400

#SPJ1

The lengths of the sides of a triangle are 4,5 and 6 if the length of the longest side of a similar triangle is 15,what is the length of the shortest side of the triangle

Answers

Answer:

The length of the shortest side of the triangle is 10.

Step-by-step explanation:

Given that the lengths of the sides of a triangle are 4, 5 and 6, if the length of the longest side of a similar triangle is 15, to determine what is the length of the shortest side of the triangle, the following calculation must be performed :

6 = 15

4 = X

4 x 15/6 = X

10 = X

Therefore, the length of the shortest side of the triangle is 10.

PLS HELP ME ON THIS QUESTION I WILL MARK YOU AS BRAINLIEST IF YOU KNKW THE ANSWER!!

Answers

Answer: will be D

Step-by-step explanation:

which of the following represents…

Answers

The correct answer is the first option

What is an equation of the line that passes through the points (0, 8) and (-8, -4)?

Answers

Answer:

y = 1.2x + 2

Step-by-step explanation:

slope = (8 - 2)/(5 - 0) = 6/5y-intercept = (0, 2)y = (6/5)x + 2y = 1.2x + 2

Answer:

y=1.5x +8

Step-by-step explanation:

Let the lengths of each side of △ABC having area equal to 1 be as follows: AB = 2, BC = a and CA = b. Let CD be a perpendicular line from point C to AB. Answer the following questions.
(1) Given AD = x, write a²+(2√3-1)b² in the form of x.
(2) Find the value of x at which a²+(2√3 - 1)b² is the lowest and the magnitude of ∠BAC.
Need help! Please show your work too. Thanks!

Answers

Answer:

Part 1)

[tex]\displaystyle \left((2-x)^2 + 1)\right) + (2\sqrt{3} - 1 ) \left(x^2 + 1\right)[/tex]

Or simplified:

[tex]\displaystyle = 2\sqrt{3}x^2 - 4x + 4 + 2\sqrt{3}[/tex]

Part 2)

The value of x for which the given expression will be the lowest is:

[tex]\displaystyle x = \frac{\sqrt{3}}{3}\approx 0.5774[/tex]

And the magnitude of ∠BAC is 60°.

Step-by-step explanation:

We are given a ΔABC with an area of one. We are also given that AB = 2, BC = a, and CA = b. CD is a perpendicular line from C to AB.

Please refer to the diagram below.

Part 1)

Since we know that the area of the triangle is one:

[tex]\displaystyle \frac{1}{2} (2)(CD) = 1[/tex]

Simplify:

[tex]\displaystyle CD = 1[/tex]

From the Pythagorean Theorem:

[tex]\displaystyle x^2 + CD^2 = b^2[/tex]

Substitute:

[tex]x^2 + 1 = b^2[/tex]

BD will simply be (2 - x). From the Pythagorean Theorem:

[tex]\displaystyle (2-x)^2 + CD^2 = a^2[/tex]

Substitute:  

[tex]\displaystyle (2-x)^2+ 1 = a^2[/tex]

We have the expression:

[tex]\displaystyle a^2 + (2\sqrt{3} - 1) b^2[/tex]

Substitute:

[tex]\displaystyle = \boxed{\left((2-x)^2 + 1)\right) + (2\sqrt{3} - 1 ) \left(x^2 + 1\right)}[/tex]

Part 2)

We can simplify the expression. Expand and distribute:

[tex]\displaystyle (4 - 4x + x^2 + 1)+ (2\sqrt{3} -1)x^2 + 2\sqrt{3} - 1[/tex]

Simplify:

[tex]\displaystyle = ((2\sqrt{3} -1 )x^2 + x^2) + (-4x) + (4+1-1+2\sqrt{3})[/tex]

Simplify:

[tex]\displaystyle = 2\sqrt{3}x^2 - 4x + 4 + 2\sqrt{3}[/tex]

Since this is a quadratic with a positive leading coefficient, it will have a minimum value. Recall that the minimum value of a quadratic always occur at its vertex. The vertex is given by the formulas:

[tex]\displaystyle \text{Vertex} = \left(-\frac{b}{2a}, f\left(-\frac{b}{2a}\right)\right)[/tex]

In this case, a = 2√3, b = -4, and c = (4 + 2√3).

Therefore, the x-coordinate of the vertex is:

[tex]\displaystyle x = -\frac{(-4)}{2(2\sqrt{3})} = \frac{1}{\sqrt{3}} =\boxed{ \frac{\sqrt{3}}{3}}[/tex]

Hence, the value of x at which our expression will be the lowest is at √3/3.

To find ∠BAC, we can use the tangent ratio. Recall that:

[tex]\displaystyle \tan \theta = \frac{\text{opposite}}{\text{adjacent}}[/tex]

Substitute:

[tex]\displaystyle \tan \angle BAC = \frac{CD}{x}[/tex]

Substitute:

[tex]\displaystyle \tan \angle BAC = \frac{1}{\dfrac{\sqrt{3}}{3}} = \sqrt{3}[/tex]

Therefore:

[tex]\displaystyle\boxed{ m\angle BAC = \arctan\sqrt{3} = 60^\circ}[/tex]

What is the value of the expression below when z=10z=10 and w=4w=4? 2z-3w 2z−3w

Answers

Answer:

8

Step-by-step explanation:

more details

2(10)-3(4)

20-12

8

Enter the location of the point as an ordered pair.
5
-5
-6

Answers

Answer:

(4,1)

Step-by-step explanation:

(4,1) is the correct answer. Answered by Gauthmath

Other Questions
When heated, carboxylic salts in which there is a good leaving group on the carbon beta to the carbonyl group undergo decarboxylation/elimination to give an alkene. Draw the structures of the products expected when this compound is heated. By selling a radio for $8400 a dealer gained 12% .how much money did she gain This prominent nun was authorized to write theological books and documented hervisions in illuminated manuscripts:O HeloiseO Galla PlacidiaO Hildegard of BingenO Theodora asap please help! and explain how you got the answer! Solve the formula A = lw for / Within the context of the individual as the basic unit of a social organization, what is a manifestation of the emphasis on individualism in many Western societies Identifique el sujeto y predicado en las siguientes oraciones Las mariposas vuelan por el jardn. El columpio se rompi el ao pasado. El helado de frambuesa era exquisito. En la casa de los vecinos, hubo una fiesta anoche. Marcos, Mara y Lucas son amigos desde hace muchos aos. Kathy receives her real estate license. Within a month of receiving her license, she gets a new job offer she cant refuse. She decides to continue to hold a license but not practice or do any real estate work or collect any referrals at this time. She is willing to renew her license, do her continuing education, and pay the same fee for her license. What should Kathy do?Seek an involuntary inactive license. Which of the following is evidence for continental drift? What is the solution to the system of equations graphed below? PLSSS HELP 8x - 126x + 8x = [?] You and a coworker seem to have a hard time seeing eye - to - eye on a number of different things . Today , she does something that makes you especially angry . What should you do? a) Tell your boss that you can't work with her anymore . b) Talk to your coworkers about how much she is driving you crazy. c) Scream at her that you've had enough of her behavior. d ) Walk away until you can speak with her calmly about the issues you have been having You need to produce a buffer solution that has a pH of 5.50. You already have a solution that contains 10 mmol (millimoles) of acetic acid. How many millimoles of acetate (the conjugate base of acetic acid) will you need to add to this solution? Translate We won't drive tomorrow into Spanish in the box below: khai niem hinh cat don gian ? Thorazine is available in a strength of 25 mg/mL. Express this strength as a percent. Sameera purchased 3 (1/2) kg of apples and 4 (3/4) kg of oranges. What is the total weight of fruits purchased by her ? give one use of zinc I dont understand what A is trying to ask please could someone help me? Thank you! Why is little being done about global warming